Laplace Transforms: Transfer Functions and Impulse

Click For Summary
The discussion revolves around solving a homework problem involving Laplace transforms, transfer functions, and impulse functions. The transfer function is identified as H(s) = 1/(75s + 0.0046), and the Laplace transforms of both the impulse and unit step functions are correctly derived. There is a query regarding the modeling of a wind force function, f_wind(t), which is constant at 4.5N for a specific time interval, raising concerns about its significance compared to other forces. The user seeks assistance with partial fraction decomposition and plotting the combined force function, which includes Dirac delta functions at specified times. The conversation emphasizes the importance of accurate modeling and understanding of the mathematical concepts involved.
ConnorM
Messages
77
Reaction score
1

Homework Statement


I uploaded the question as a picture and attached it.

Homework Equations


Unit step function -
u_c (t) =<br /> \begin{cases}<br /> 1 &amp; \text{if } t \geq c \\<br /> 0 &amp; \text{if } t &lt; c<br /> \end{cases}

Impulse function -
δ(t) = \displaystyle\lim_{Δ\rightarrow 0} δ_Δ (t)

Multiplication Property for Impulse function -
f(t)⋅δ(t - t_d) = f(t_d)⋅δ(t - t_d)
*A function f(t) becomes a value f(t_d)*

The Attempt at a Solution



(a and b)

I have determined that both of the transfer functions are the same,

H(s) = V(s)/F_{p / w}(s) = {\frac{1}{75s + 0.0046}}

(c)

The Laplace transform of the impulse function is 1 so,

V(s) = {\frac{1}{75s + 0.0046}}

v(t) = {\frac{1}{75}} e^{{\frac{-0.0046}{75}}t}

(d)

The Laplace transform of the unit step function is 1/s so,

V(s) = {\frac{1}{s(75s + 0.0046)}}

v(t) = 217.391 - 217.391 e^{{\frac{-0.0046}{75}}t}

***Am I right up to this point?***

(e)

f_{wind} (t) =<br /> \begin{cases}<br /> 4.5 &amp; \text{if }1 \geq t &lt; 10 \\<br /> 0 &amp; \text{otherwise,} <br /> \end{cases}

Does that mean that from 1 -> 10 there is a constant force of only 4.5N? That just seems negligible compared to the force applied by the skaters pushes.

f(t) = f_{wind}(t) + 1160δ(t-4) + 935δ(t-6) + 708δ(t-7.8)

Not quite sure how to model this!
 

Attachments

  • Assignment1-Q5.JPG
    Assignment1-Q5.JPG
    48.1 KB · Views: 492
Physics news on Phys.org
ConnorM said:

Homework Statement


I uploaded the question as a picture and attached it.

Homework Equations


Unit step function -
u_c (t) =<br /> \begin{cases}<br /> 1 &amp; \text{if } t \geq c \\<br /> 0 &amp; \text{if } t &lt; c<br /> \end{cases}

Impulse function -
δ(t) = \displaystyle\lim_{Δ\rightarrow 0} δ_Δ (t)

Multiplication Property for Impulse function -
f(t)⋅δ(t - t_d) = f(t_d)⋅δ(t - t_d)
*A function f(t) becomes a value f(t_d)*

The Attempt at a Solution



(a and b)

I have determined that both of the transfer functions are the same,

H(s) = V(s)/F_{p / w}(s) = {\frac{1}{75s + 0.0046}}

(c)

The Laplace transform of the impulse function is 1 so,

V(s) = {\frac{1}{75s + 0.0046}}

v(t) = {\frac{1}{75}} e^{{\frac{-0.0046}{75}}t}

(d)

The Laplace transform of the unit step function is 1/s so,

V(s) = {\frac{1}{s(75s + 0.0046)}}

v(t) = 217.391 - 217.391 e^{{\frac{-0.0046}{75}}t}
Assuming units of m/sec, this seems way too large. Can you show what you did in going from V(s) to v(t)? There's a partial fractions decomposition involved.
ConnorM said:
***Am I right up to this point?***

(e)

f_{wind} (t) =<br /> \begin{cases}<br /> 4.5 &amp; \text{if }1 \geq t &lt; 10 \\<br /> 0 &amp; \text{otherwise,}<br /> \end{cases}
You have a typo: The first case restriction is ##1 \le t < 10##
ConnorM said:
Does that mean that from 1 -> 10 there is a constant force of only 4.5N? That just seems negligible compared to the force applied by the skaters pushes.

f(t) = f_{wind}(t) + 1160δ(t-4) + 935δ(t-6) + 708δ(t-7.8)

Not quite sure how to model this!
 
I can't write them out right now since I'm on my phone. I just checked them on wolfram alpha and it gave the same answer.
 
ConnorM said:

Homework Statement


I uploaded the question as a picture and attached it.

Homework Equations


Unit step function -
u_c (t) =<br /> \begin{cases}<br /> 1 &amp; \text{if } t \geq c \\<br /> 0 &amp; \text{if } t &lt; c<br /> \end{cases}

Impulse function -
δ(t) = \displaystyle\lim_{Δ\rightarrow 0} δ_Δ (t)

Multiplication Property for Impulse function -
f(t)⋅δ(t - t_d) = f(t_d)⋅δ(t - t_d)
*A function f(t) becomes a value f(t_d)*

The Attempt at a Solution



(a and b)

I have determined that both of the transfer functions are the same,

H(s) = V(s)/F_{p / w}(s) = {\frac{1}{75s + 0.0046}}

(c)

The Laplace transform of the impulse function is 1 so,

V(s) = {\frac{1}{75s + 0.0046}}

v(t) = {\frac{1}{75}} e^{{\frac{-0.0046}{75}}t}

(d)

The Laplace transform of the unit step function is 1/s so,

V(s) = {\frac{1}{s(75s + 0.0046)}}

v(t) = 217.391 - 217.391 e^{{\frac{-0.0046}{75}}t}

***Am I right up to this point?***

(e)

f_{wind} (t) =<br /> \begin{cases}<br /> 4.5 &amp; \text{if }1 \le t &lt; 10 \\<br /> 0 &amp; \text{otherwise,}<br /> \end{cases}

Does that mean that from 1 -> 10 there is a constant force of only 4.5N? That just seems negligible compared to the force applied by the skaters pushes.

f(t) = f_{wind}(t) + 1160δ(t-4) + 935δ(t-6) + 708δ(t-7.8)

Not quite sure how to model this!

Ok I have fixed the function, f_{wind}(t).

For,

V(s) = {\frac{1}{s(75s + 0.0046)}}

Partial Fraction Decomp.

V(s) = A/s + B/(75s + 0.0046)

1 = A(75s + 0.0046) + B(s)

subbing in s = 0,

A = 1/0.0046 = 217.391

subbing in s = -0.0046/75

1 = B(-0.0046/75)

B = -75/0.0046

V(s) = 217.391/s + (-75/0.0046)/(75s + 0.0046)

v(t) = 217.391 - 217.391e^{-0.0046/75}

Could someone help me with part 5e? I'm not sure how to plot this.
 
Here is what I have so far,

f(t) = 1160dirac(t-4) + 935dirac(t-6) + 708dirac(t-7.8) + 4.5(u(t-1) - u(t-10))
 
Question: A clock's minute hand has length 4 and its hour hand has length 3. What is the distance between the tips at the moment when it is increasing most rapidly?(Putnam Exam Question) Answer: Making assumption that both the hands moves at constant angular velocities, the answer is ## \sqrt{7} .## But don't you think this assumption is somewhat doubtful and wrong?

Similar threads

  • · Replies 10 ·
Replies
10
Views
2K
  • · Replies 1 ·
Replies
1
Views
1K
  • · Replies 1 ·
Replies
1
Views
1K
  • · Replies 4 ·
Replies
4
Views
3K
  • · Replies 3 ·
Replies
3
Views
3K
Replies
3
Views
2K
  • · Replies 3 ·
Replies
3
Views
1K
  • · Replies 4 ·
Replies
4
Views
3K
  • · Replies 1 ·
Replies
1
Views
1K
  • · Replies 8 ·
Replies
8
Views
2K